Beweis - stammfunktionen

Neue Frage »

kingskid Auf diesen Beitrag antworten »
Beweis - stammfunktionen
Hi!
Wie kann man beweisen, dass sich zwei Stammfunktionen einer Funktion nur um eine Konstante unterscheiden, also dass
F(x) = G(x) + c ist?

dachte irgendwie an den mittelwertsatz, aber wie das genau funktioniert ist mir nicht klar...

viele grüße
MrPSI Auf diesen Beitrag antworten »

Leite doch mal deine Gleichung ab. Augenzwinkern

Ansonsten findest du bei Mathe-online einen Beweis.
Frooke Auf diesen Beitrag antworten »

Vermutlich versteh ich Dich falsch, aber definitionsgemäß ist ja



und wegen



EDIT: Um Sekunden zu spät.
JochenX Auf diesen Beitrag antworten »

Zitat:
Original von Frooke


und wegen


ja, das reicht tatsächlich nicht - damit zeigst du: ist F Stammfkt, dann auch F+c.

Allerdings zeigst du noch nicht, dass eine weitere Stammfunktion von der Bauart F+c sein muss.


Den Schritt schafft man dann mit Ableiten der Differenzenfunktion auch gerade noch.
Frooke Auf diesen Beitrag antworten »

Ja, da war ich zu ungenau. Tut mir leid. unglücklich
kingskid Auf diesen Beitrag antworten »

hi!
danke für den link, is ne coole seite hab nur den beweis nicht gefunden...??

wie meinst du das mit ableiten der differenzenfunktion @loed??

viele grüße
 
 
Ben Sisko Auf diesen Beitrag antworten »

Mit "Differenzenfunktion" meint er die Differenz der zwei Stammfunktionen.

Also:"Sei F eine Stammfunktion. Nimm an, G ist eine weitere Stammfunktion,..."
kingskid Auf diesen Beitrag antworten »

d.h. so:

F(x) - G(x) = c ???

und dann??
Ben Sisko Auf diesen Beitrag antworten »

Ne, erstmal nur F(x)-G(x). Das dann ableiten, was kommt raus?
kingskid Auf diesen Beitrag antworten »

hmm, meinst du ich soll das mit diesem grenzwert ableiten?



und da für f(x) F(x)-G(x) irgendwie einsetzen??
klarsoweit Auf diesen Beitrag antworten »

Nein. Du sollst die Funktion H(x) := F(x)-G(x) ableiten. Was kommt da raus?
kingskid Auf diesen Beitrag antworten »

sorry, irgendwie check ich das grad nicht... mit was und wie soll ich das machen??
klarsoweit Auf diesen Beitrag antworten »

Also das ganze nochmal von vorn:
Zu einer Funktion f hast du eine Stammfunktion F(x) und noch eine weitere Stammfunktion G(x). Jetzt bilden wir die Funktion H(x) := F(x) - G(x). Wie sieht dann die Ableitung H'(x) aus?
Frooke Auf diesen Beitrag antworten »

Übrigens findet man den Beweis, indem man rechts auf «Beweis» klickt smile ...
kingskid Auf diesen Beitrag antworten »

achsoo, das ist einfach

H'(x) = F'(x) - G'(x) = f(x) - f(x) = 0

d.h. H(x) muss eine Konstante sein...?
also H(x) = F(x) - G(x) = c, unterscheiden sich also um c.


aber wozu braucht man dann den mittelwertsatz??
klarsoweit Auf diesen Beitrag antworten »

Den braucht man für den Beweis, daß H(x) eine Konstante c.

Beachte: daß die Ableitung einer Konstanten Null ergibt, ist klar.
Aus der Tatsache, daß die Ableitung einer Funktion immer Null ist, muß nicht zwangsläufig folgen, daß die uresprüngliche Funktion eine Konstante war.
MrPSI Auf diesen Beitrag antworten »

Zitat:

Beachte: daß die Ableitung einer Konstanten Null ergibt, ist klar.
Aus der Tatsache, daß die Ableitung einer Funktion immer Null ist, muß nicht zwangsläufig folgen, daß die uresprüngliche Funktion eine Konstante war.

Kannst du bitte dazu mal ein Beispiel geben .... verwirrt
kingskid Auf diesen Beitrag antworten »

@klarsoweit: danke für die erklärung, aber wie setzt man den mittelwertsatz dann konkret ein?

ich mein der sagt doch, dass wenn f stetig diffbar ist auf Intervall[a,b] dann existiert ein c aus [a,b] mit

oder?

wie kann ich damit zeigen dass H(x) eine Konstante sein muss?

und ja, so ein bsp würde mich auch interessieren, was für nicht konstante funktionen gibt es deren ableitung 0 ist?

viele grüße
klarsoweit Auf diesen Beitrag antworten »

Zitat:
Original von kingskid
ich mein der sagt doch, dass wenn f stetig diffbar ist auf Intervall[a,b] dann existiert ein c aus [a,b] mit

oder?

wie kann ich damit zeigen dass H(x) eine Konstante sein muss?

Angenommen die Funktion H wäre nicht konstant. Dann wähle man die Punkte a und b so, daß H(a) <> H(b). Und damit gibt es ein c mit H'(c) <> 0.

Zitat:
Original von MrPSI
Kannst du bitte dazu mal ein Beispiel geben .... verwirrt

Kann ich natürlich nicht. Es wird ja gerade bewiesen, daß die Funktion eine Konstante sein muß. Was ich sagen wollte, ist folgendes:

In der Aussage
darf man nicht einfach den Pfeil umdrehen, nur weil man kein passendes Gegenbeispiel findet. Das sollte schon ordentlich bewiesen werden. Augenzwinkern
irre.flexiv Auf diesen Beitrag antworten »

Man kann es auch direkt beweisen. Ist auf

Nun wähle man mit . Dann gilt auf bzw. der Mittelwertsatz

kingskid Auf diesen Beitrag antworten »

okay, dankeschön für eure erklärungen smile
Neue Frage »
Antworten »



Verwandte Themen

Die Beliebtesten »
Die Größten »
Die Neuesten »